BLW FINAL EXAM REVIEW

Ace your homework & exams now with Quizwiz!

Edible Arrangements sued a competitor, 1-800 Flowers.com claiming trademark infringement based on the competitor's use of the term 'bouquet' for its gift baskets containing fresh cut fruit products. Under these circumstances, which of the following is true? Answers: 1-800 Flowers.com can make a strong argument that the term 'bouquet' is a generic term that is not subject to trademark protection. If 1-800 Flowers.com can prove that it did not intend to confuse customers about the origin of its products, it will win this infringement lawsuit. All that Edible Arrangements has to prove to win this infringement lawsuit is that it was the first business in interstate commerce to use the term 'bouquet' for gift baskets containing fresh cut fruit products. Since facts are not subject to trademark protection, Edible Arrangements will lose its infringement lawsuit.

1-800 Flowers.com can make a strong argument that the term 'bouquet' is a generic term that is not subject to trademark protection.

Buyer and Seller enter into a signed written enforceable contract for the sale of 1,000 umbrellas @ $5.00. Buyer breaches the contract and Seller finds a substitute buyer who buys the umbrellas for $4.00 per umbrella (a reasonable price under all the circumstances). What damages, if any, is Seller entitled to recover from Buyer? Answers: $1,000 $5,000 $4,000 $0

$1,000

Seller enters into a signed written enforceable contract with Buyer for the sale of a classic sports car for $12,000. Assume Seller breaches the contract by refusing to sell the car. Buyer is able to find a comparable sports car which he buys for $13,000. What damages can Buyer recover from Seller? Answers: $13,000 $12,000 $1,000 $0

$1,000

Homeowner has entered into a contract with contractor to build a swimming pool in her backyard for a total price of $45,000 (inclusive of all labor, materials and builder profit). The builder budgeted $35,000 in construction costs and anticipated a profit of $10,000 for this project. Assume that homeowner breaches the contract by notifying contractor before he started work (and before he contracted for any materials or labor for the project) that homeowner did not want the pool any longer because he just learned that he's been laid off from his job. What compensatory damages, if any, can contractor recover from homeowner? Answers: $10,000 $45,000 $35,000 $0

$10,000

Chloe drove through an intersection without looking and hit Mike's car that he had driven into the intersection without obeying a stop sign. Chloe sued Mike. The jury found that Mike was negligent, and that Chloe was also negligent with Chloe's allocated fault equal to 30% and Mike's allocated fault equal to 70%. The jury further found that Chloe's total damages were $200,000. Under comparative negligence, what is the amount of damages Chloe can recover from Mike? Answers: $60,000 $140,000 $0 $200,000

$140,000

Charlie the Contractor enters into a signed written contract to build an addition on Harry Homeowner's home for $20,000. Charlie begins the job but stops work after the first week without any valid excuse. There is no indication that Charlie will complete the job and Charlie does not return any of Harry's phone calls. Harry paid Charlie $7,000 up-front before the work started. Harry hires Bob, his brother, to finish the addition at a contract price of $20,000. However, Harry turned down three other quotes from reputable contractors to finish the job. All three of these other quotes were for a contract price of $16,000. Assuming that Charlie is in breach of contract, what amount of damages, if any, is Harry entitled to recover from Charlie based on these facts? Answers: $7,000 $13,000 $11,000 $3,000

$3,000

Seller was trying to sell his house for $250,000. On January 5, 2012 Buyer went to the Post Office and sent Seller a signed written offer to buy seller's house for $235,000 (which Seller received on January 8, 2012 ). On January 9, 2012 Seller went to the Post Office and sent a letter to Buyer stating that Seller was willing to sell the house to Buyer for $245,000 (which Buyer received January 11, 2012). On January 12, 2012 Buyer went to the Post Office and sent seller a letter stating his agreement to buy the house for $245,000 (which seller received on January 14, 2012) Under these circumstances, which of the following is true:

Buyer and Seller entered into a contract on January 12, 2012

Which of the following is currently subject to copyright protection? Answers: The idea behind the system for double-entry bookkeeping. Shakespeare's play Romeo and Juliet. The fact that there are 198 lakes and ponds in Connecticut. An original short story written last year by Prof. Smith.

An original short story written last year by Prof. Smith.

If a female applicant can prove that she was rejected by the City for a position as a firefighter solely because she is female, what type of legal claim, if any, could she make against the City? Answers: None. Under the doctrine of freedom of contract, the City has complete discretion in who it chooses to hire or fire. A claim of employment discrimination based on disparate treatment. A claim of res judicata. A claim of employment discrimination based on res ipsa loquitor.

A claim of employment discrimination based on disparate treatment.

Which of the following contracts is governed by Article 2 of the Uniform Commercial Code? Answers: A contract to clean Jill's house. A contract to buy an antique Ford Edsel car. A contract to sell a farmhouse. A contract to provide Internet marketing services. A contract to lease a sandwich shop.

A contract to buy an antique Ford Edsel car.

Which of the following contracts is not required to be in writing under the Statute of Frauds?

A contract to buy an iPad for $350

A man with 10 years of experience as a waiter applied to be a food server at a Hooters Restaurant. Hooters refused to consider him for this position because he was male. If the applicant sued Hooters for employment discrimination which of the following statement(s) is/are true? Answers: Hooter's would likely raise the bona fide occupational qualification defense in this employment discrimination lawsuit. The legal basis for the plaintiff's claim is disparate treatment under Title VII of the Civil Rights Act of 1964. Because it has more than 15 employees, Hooters is subject to Title VII of the Civil Rights Act of 1964. All of the above.

All of the above.

Steve slips on a broken bottle of ketchup in the local WalMart. In order to prevail in a claim of negligence against WalMart, Steve will be required to prove: Answers: The store owed Steve a duty of care. The store failed to exercise reasonable care under all the circumstances by allowing the ketchup to be on the floor. It was reasonably foreseeable that an injury such as this could occur under these circumstances Steve suffered an injury as a result of the store's failure to exercise reasonable care. All of the above.

All of the above.

What are the elements of a cause of action for fraudulent misrepresentation? Answers: A false statement of material fact Intent to deceive Justifiable reliance All of the above.

All of the above.

Which of the following is an illegal contract? Answers: A contract for a loan at a usurious rate of interest. A fraternity betting pool on the outcome of the World Series A life insurance policy taken out on the life of someone when there is no insurable interest. All of the above.

All of the above.

Which of the following scenarios is an example of copyright infringement: Answers: Prof. McNamara is very impressed with a textbook written by Barbara Sullivan. To supplement the textbook McNamara is currently using in her college course, the professor makes 100 copies of five chapters of Sullivan's eight-chapter book. The professor distributes these copies to her 100 students. Martha borrows a CD from the public library and burns a copy of the CD to her laptop. Jake uses a free peer to peer file-sharing website to download 20 tracks of his favorite Beatles' music onto his MP3 player. All of the above.

All of the above.

Which of the following statements accurately describes a traditional ethical theory? Answers: Under the ethics of character/virtue, ethical conduct is that taken by a person of high moral character who acts accordingly. Under the ethics of utilitarianism, ethical conduct is that which creates the greatest good for the greatest number of people. Under the ethics of deontology, ethical conduct is that which would be a good thing if everyone did it. All of the above.

All of the above.

Which of the following statements is/are true under current American copyright law? Answers: A copyright lasts for 70 years after the death of the author. Once a copyright expires, the work is in the public domain so anyone can use it without permission of the copyright holder. Copyright protection arises automatically when an original work is fixed in a tangible medium. All of the above.

All of the above.

Which of the following uses would likely be permitted under the Fair Use Doctrine? Answers: Limited use of copyrighted material in news reporting. Limited use of copyrighted material for scholarship. Limited use of copyrighted material for teaching provided the use is brief and spontaneous. All of the above.

All of the above.

Which of the following contracts is governed by Article 2 of the Uniform Commercial Code?

Contract to buy a steinway piano

Which of the following is NOT a protected category under Title VII of the Civil Rights Act of 1964? Answers: Race Sex Educational level Religion

Educational level

Which of the following statements is/are true: Answers: Failure to follow corporate formalities may subject stockholders in a corporation to personal liability. A sole proprietorship is harder to form than a corporation. A partnership provides protection from personal liability to the partners. All of the above

Failure to follow corporate formalities may subject stockholders in a corporation to personal liability.

In 2014, Melissa developed a hydrogen powered engine for a car while working in her garage in her spare time. Melissa installed the engine in a small car that she has been driving around town. In 2015, engineers working for Ford Motor Co. independently developed an identical hydrogen powered engine. After completing tests on the engine for two years, Ford filed a patent for the engine on March 1, 2017. On March 2, 2017, Melissa also filed for a patent for her engine. Assuming the engine satisfies all of the requirements for a patent, who is entitled to the patent? Answers: Ford, but only if Ford is the first to sell the engine Ford, because Ford was the first to file for the patent Melissa, because Melissa was the first to invent the engine and put it into practice Both Melissa and Ford must share in the patent rights

Ford, because Ford was the first to file for the patent

What is the general goal of awarding compensatory damages in a breach of contract situation?

Placing the non breaching party in the same position as if the contract had been fully performed.

Before Edward Miller left his job as a salesperson at the New England Insurance Agency, Inc., he made a copy of New England's customer lists. He used these lists to successfully solicit many of New England's customers to move to his new employer. At New England, the customer lists had been kept in file cabinets. Although the company did not restrict access to these files, it said there was an understanding to the effect that "you do not peruse my files and I do not peruse yours." The lists were not marked "confidential" or "not to be disclosed." New England sued Miller for stealing its trade secrets. Miller's strongest argument is that Answers: His freedom of contract entitled him to switch employers New England did not take reasonable efforts to safeguard the customer lists New England was not harmed by his actions Customer list are simply a collection of facts and therefore not subject to intellectual property protection

New England did not take reasonable efforts to safeguard the customer lists

On February 1, Barbara Buyer entered into a legally binding enforceable written contract with Suzy Seller agreeing to buy Suzy's 1994 red Mazda Miata on February 15 for $2,500. On February 3, Suzy sends Barbara an e-mail advising Barbara that she has changed her mind and will not be selling her Miata to Barbara. Which of the following statements is/are true? Answers: Since Suzy notified Barbara of the cancellation of the contract within 3 business days, Suzy is not in breach of contract. Since Barbara has not paid Suzy a deposit, Suzy can cancel the contract at any time. If Barbara can find a substantially similar 1994 red Mazda Miata to buy for $2,500 from someone else and if she has not incurred any expenses as a result of Suzy's change of mind then Barbara is not entitled to collect any damages from Suzy (except possibly nominal damages). All of the above.

If Barbara can find a substantially similar 1994 red Mazda Miata to buy for $2,500 from someone else and if she has not incurred any expenses as a result of Suzy's change of mind then Barbara is not entitled to collect any damages from Suzy (except possibly nominal damages).

Which of the following is an advantage of a corporation? Answers: It is easy to form a corporation. It requires little expense to form a corporation. It offers limited liability for its shareholders. It is a useful estate planning tool.

It offers limited liability for its shareholders.

Lucky golden retriever has recently had a litter of puppies. Marry has agreed to buy the last remaining puppy for $350 and they agree that Mary will pick up the puppy and pay for the puppy on Friday. Mary calls Lucy on Thursday to tell her she has changed her mind and won't buy the puppy after all. When Lucy's friend Frieda overhears this phone conversation, Frieda immediately tells Lucy that she'll buy the puppy herself. If Lucy sues Mary for breach of contract, which of the following is true:

Lucy can not recover any damages from Mary under these circumstances.

XYZ Corp. hired Joe to bribe a foreign government official into awarding a $3 million contract to XYZ, XYZ gave Joe $10,000 in cash to make the bribe payment and $2,000 for Joe's efforts. Instead of paying the bribe, Joe pocketed all of the money. If XYZ sues Joes, XYZ will:

Not be able to recover the $12,000

Which of the following marks is a fanciful trademark? Answers: Nyquil for the cold medicine Prince for a tennis racket. Best Buy for a chain of electronics store. Blue Diamond for nuts.

Nyquil for the cold medicine

Which of the following is NOT an established category of protected trademarks? Answers: Suggestive Res ipsa loquitor Arbitrary Fanciful

Res ipsa loquitor

Which of the following statements is/are true regarding the structure of a business? Answers: Structuring a business as a sole proprietorship provides protection from personal liability to the owner. Structuring a business as a general partnership provides protection from personal liability to the partners. Structuring a business as a limited liability company provides protection from personal liability to the members. All of the above

Structuring a business as a limited liability company provides protection from personal liability to the members

Milton Friedman would have been most likely to agree with which of the following statements? Answers: The CEO and the board of directors of a corporation should satisfy the needs of all the corporation's stakeholders. The CEO and the board of directors should focus on delivering maximum value to the shareholders of the business while complying with the law and with ethical customs. The CEO and the board of directors should be focused on the triple bottom line - people, profits and the planet. All of the above.

The CEO and the board of directors should focus on delivering maximum value to the shareholders of the business while complying with the law and with ethical customs.

A knife manufacturer was sued in a claim of strict product liability by a consumer who cut his hand while slicing bread with the knife. The injured consumer claims that the manufacturer failed to adequately warn him that the knife was sharp. What defense would you raise if you were the knife manufacturer under these facts? Answers: The defendant is not liable because the fact that knives are sharp is a generally known danger so there is no need to warn of it. Under the doctrine of res ipsa loquitor, the defendant is not liable. The defendant is not liable because this injury is an unforeseen circumstance. All of the above

The defendant is not liable because the fact that knives are sharp is a generally known danger so there is no need to warn of it.

Mary Jones is expanding her marketing firm (which currently has 20 employees). Mary places the following ad on Monster.com and other job applicant websites: "Successful Marketing Firm seeks to hire marketing associate with demonstrated skills in Internet keyword advertising campaigns. Job Requirements: college degree in marketing or related field, minimum of 5 years of relevant work experience. Only females will be considered for this position." Which of the following statements best describes the application of Title VII of the Civil Rights Act of 1964 ("Title VII") to this hiring practice? Answers: This hiring practice is legal under Title VII as it is prima facie non-discriminatory. This hiring practice is legal because being female is essential to this position. This hiring practice is illegal discrimination based on disparate impact. This hiring practice is illegal discrimination based on disparate treatment.

This hiring practice is illegal discrimination based on disparate treatment.

Which of the following statements is/are true: Answers: Immanuel Kant was a German philosopher who was a proponent of utilitarian ethics. Under the Shareholder Primacy Theory, the corporation owes a duty to all the company's stakeholders and has a duty to contribute positively to the world around it. Under utilitarian ethics, if a decision creates the greatest good for the greatest number of people, it is ethical. All of the above

Under utilitarian ethics, if a decision creates the greatest good for the greatest number of people, it is ethical.

Which of the following types of property would be considered "goods" under the Uniform Commercial Code?

a computer

According to the stakeholder theory: Answers: a socially responsible corporation should consider the impact of its decision on the shareholders only a corporation's social responsibilities are limited to staying within the boundaries of the law when it complies with its legal duties, the corporation can ignore any other social obligations all the stakeholders in a corporate decision deserve some kind of moral consideration

all the stakeholders in a corporate decision deserve some kind of moral consideration

An injunction is a(n) _____. Answers: equitable doctrine intended to equalize the position of two non-breaching parties court order directing a person to stop doing that which he/she should not do restoring to one party of what he gave to the other award for the purpose of punishing a defendant in a civil action

court order directing a person to stop doing that which he/she should not do

Sally offered to sell her bicycle to Bob for $100. Bob said that $100 was too much, but that he would give her $75. Sally said no. Then, Bob said that he would pay Sally the entire $100. Which of the following is/are true?

B and C (Bob's statement that he would give Sally $75 operated as a rejection of Sally's offer by Bob

Which of the following is not a protected/ distinctive category of trademark?

Generic Mark

If a manufacturer produces a defective product, sells it to a wholesaler, who sells it to a retail who sells it to a consumer, and the consumer is injured, who is/are potentially liable to the consumer under strict product liability?

The manufacturer, wholesaler, and retailer

he remedy of specific performance may be available for the breach of a contract to sell: Answers: a beach cottage A used 2019 Prius car A set of new Pottery Barn bedroom furniture All of the above

a beach cottage

The business form that offers the limited liability of a corporation and the tax status of a pass-through entity is Answers: a close corporation. a sole proprietorship. a limited liability company. a general partnership.

a limited liability company.

The burden of proof required in a criminal case is: Answers: highest degree of honesty. clear and convincing. a preponderance of the evidence. beyond a reasonable doubt.

beyond a reasonable doubt.

Judith is a CPA with an excellent reputation and client base. She sells her tax preparation business to Shawn, and the sales contract includes a noncompete clause restricting Judith from opening a similar business for one year within a 10-mile radius of her former office. If she opens a tax preparation office six months after selling her business to Shawn which new office is located five miles away from her former office, a court would probably: Answers: order confiscation of all of her new client files and turn them over to Shawn. refuse to enforce the noncompete clause, as it is unreasonable regarding time and geographical area. enforce the non-compete agreement. refuse to become involved, as the noncompete clause was illegal.

enforce the non-compete agreement.

Common law refers to Answers: law made by legislatures in the form of statutes. law made when judges decide cases and then follow those decisions in later cases. law that is the same or similar in all the states. the legal systems of France, Germany, and Italy.

law made when judges decide cases and then follow those decisions in later cases.

Ryan, a minor, contracted to sell his auto to Ed, a 28-year-old. Ryan later refused to complete the sale. If Ed sues to enforce the contract, Ed will Answers: lose under the doctrine of quasi-contract. win because minors can only avoid contracts for the sale of land. lose, because the contract is voidable by Ryan. Win because the contract is executory. win because Ryan was the seller.

lose, because the contract is voidable by Ryan.

Charles and Ellen, an unmarried couple, run an ice cream store. The business is not incorporated and they have filed no formation papers with the state. Their business is a: Answers: sole proprietorship. corporation. partnership. limited liability company.

partnership.

Solomon breaches his contract with Neal to purchase the 500 pairs of socks he had promised to buy. Neal is able to sell the 500 pairs to Renny for a much lower amount. Neal then sues Solomon for damages. Neal will be able to recover Answers: the difference between Solomon's contract price and the amount paid by Renny. Solomon's contract price. an amount which depends on whether Solomon intentionally breached because he found cheaper socks somewhere else. Any of the above at Neal's option.

the difference between Solomon's contract price and the amount paid by Renny.

Which of the following would NOT be personally liable for the debts of the business? C. an S corp shareholder

utilitarianism

What are steps in the Markkula ethical decision-making process: Answers: Recognize an Ethical Issue & Get the Facts Evaluate Alternative Actions & Make a Decision and Test it Act and Reflect on the Outcome All of the above

All of the above

Which of the following are considered stakeholders? Answers: Employees Customers Shareholders All of the above

All of the above

Which of the following is/are not subject to patent protection? Answers: An abstract idea Laws of nature Scientific principles All of the above

All of the above

Which of the following, if proven, would result in a finding that a product was in a defective condition making it unreasonably dangerous to the consumer? Answers: Defect in the manufacture Defect in the design Defect in the packaging Inadequate instructions or warnings All of the above

All of the above

Zachary graduated from college and sold most of the contents of his apartment to Sally. What law governs the contract the parties made? Answers: real property law the Convention on Contracts the common law Article 2 of the Uniform Commercial Code none of these, as the contract is informal

Article 2 of the Uniform Commercial Code

obcat Pizza Restaurant employs Chris as a part-time delivery driver. While delivering three pepperoni pizzas to QU students living off-campus, Chris is busy checking his e-mail, updating his Facebook status and talking to his roommate on his iPhone. Chris is so distracted with all these activities that he isn't paying attention to his driving. He runs a red light and crashes into Beverly's van. Beverly is seriously injured and her van is totaled. Under these circumstances: Answers: Beverly can recover from Bobcat Pizza under the doctrine of respondeat superior. If the manager at Bobcat Pizza had warned Chris to pay careful attention to his driving, then Bobcat Pizza would not be liable to Beverly. Bobcat Pizza is liable to Beverly under the doctrine of res ipsa loquitor. All of the above.

Beverly can recover from Bobcat Pizza under the doctrine of respondeat superior.

Bill owns a retail business by himself as a sole proprietor and was sued by a customer who fell in the store. The customer claimed the business was negligent in caring for its floors. Assuming that the customer is successful in this lawsuit, which statement best describes Bill's potential liability? Answers: Bill can only be liable up to the amount he initially invested in the business. Bill cannot be held personally responsible; the customer's insurance must pay for the claim. Bill has no potential liability to the customer. Bill can be held personally liable to the customer since he is a sole proprietor.

Bill can be held personally liable to the customer since he is a sole proprietor.

Which of the following marks is an arbitrary trademark? Answers: Blackberry for the smartphone. McDonald's for the fast food chain. Nyquil for the cold medicine. Exxon for the oil company.

Blackberry for the smartphone.

Joan is at the local movie theater watching the latest summer blockbuster. After the movie, she starts walking to the lobby of the movie theater but notices that the theater across the hall is about to play another movie that she really wants to see. Joan would love to see the movie without having to buy another ticket, but she stops herself from sneaking into the theater because she knows that if everyone did this, the movie theater could go out of business. Joan's thinking is most aligned with that of: Answers: Utilitarianism Shareholder Primacy Theory Deontology Corporate Social Responsibility

Deontology

FreeFiles.net is a website that distributes free software that allows computer users to share electronic files through peer-to-peer networks. This software lets users' computers communicate directly with each other without using a central server. Hundreds of thousands of users share billions of files every month using FreeFiles.net. Over 95% of these shared files are copyrighted TV shows and movies. The holders of the copyrights to these TV shows and movies have brought suit against FreeFiles.net alleging violation of copyright law. Which of the following is true? Answers: Freefiles.net is not liable to the copyright holders under the Fair Use Doctrine. Free files.net is not violating copyright law under the First Sale Doctrine. Freefiles.net is liable to the copyright holders under secondary liability for contributory copyright infringement. FreeFiles.net is not violating copyright law because it doesn't charge users for the file-sharing software.

Freefiles.net is liable to the copyright holders under secondary liability for contributory copyright infringement.

When an appeal is filed with the US supreme court, the supreme court

Has discretion as to which case it hears

Simon has entered into a signed written contract to buy a house in Indiana for $300,000. A week before the closing is scheduled to take place for him to purchase the house, Simon learns that he is being transferred to Connecticut by his employer. He contacts the real estate agent to tell her that he doesn't want to buy this house now that he's required to move out of state. The realtor advises Simon to purchase the house and then immediately put the house back on the market. Under these circumstances which of the following statements is true: Answers: If Simon chooses not to perform his obligations under the contract, he can be held liable for damages for breach of contract. Since Simon entered into a signed written contract, his only choice is to proceed with the purchase of the house in accordance with the contract. Because Simon has a good reason for not buying the house, he can terminate the contract without any liability to the seller. If Simon chooses not to perform his obligations under the contract, he can be put in jail. Under the doctrine of freedom of contract, Simon is free to cancel this transaction at any time.

If Simon chooses not to perform his obligations under the contract, he can be held liable for damages for breach of contract.

Stuart went to Barnes and Noble and paid $15 for Justin Timberlake's newest album on CD. He knows that his friend Peter would also really like to have this music. Which of the following is true: Answers: Under the first sale doctrine, since Stuart legally purchased this music, he can do anything he wants with the CD. If Stuart listens to the CD, decides he doesn't like the music and gives the CD to Peter (without making any copies of the music), there is no violation of copyright law. If Stuart burns a copy of the CD to his laptop and then he sells the original CD to Peter for less than $1.00, there is no violation of copyright law. If Stuart burns just one song from the CD to his laptop and then he gives the original CD to Peter, there is no violation of copyright law.

If Stuart listens to the CD, decides he doesn't like the music and gives the CD to Peter (without making any copies of the music), there is no violation of copyright law.

Google regularly alerts the news media that the proper way to refer to the company is as follows: "I will perform a Google® search to find that restaurant." The following is incorrect usage: "I will google that restaurant." The reason that Google makes this effort is: Answers: There is no reason for Google to do this because once a company has registered its trademark with the Federal Patent and Trademark Office, there are no circumstances under which it can lose its rights to that trademark. All trademark holders are required by law to provide this notification to newspapers, television and radio stations once a month. If a trademark becomes so widely used that it becomes a generic term, then the company will lose its trademark protection. They are trying to generate free publicity.

If a trademark becomes so widely used that it becomes a generic term, then the company will lose its trademark protection.

Which of the following statements is/are FALSE: Answers: Compensatory damages are intended to put the non-breaching party in the same condition as if the contract had been fully performed. If the non-breaching party does not take reasonable steps to mitigate the damages suffered as a result of the breach, the court will penalize him by awarding zero compensatory damages. Specific performance is an available remedy for breach of a contract to sell a unique item such as a rare vintage car. All of the above.

If the non-breaching party does not take reasonable steps to mitigate the damages suffered as a result of the breach, the court will penalize him by awarding zero compensatory damages.

Which best describes assumption of the risk in a negligence case? Answers: The defendant is not liable if the plaintiff's injuries resulted from gambling or some other immoral activity The plaintiff is more at fault than the defendant in causing the accident. Under the common law doctrine of res ipsa loquitor, a plaintiff is presumed negligent unless the risk is assumed by the defendant. If the plaintiff knowingly and willingly subjected herself to an obvious risk this is a defense to a claim of negligence.

If the plaintiff knowingly and willingly subjected herself to an obvious risk this is a defense to a claim of negligence.

Robert started a chain of 'Sprinkles' cupcake bakeries in California that also sells its 'Sprinkles' products in national chain stores including Williams-Sonoma. (There are several Williams-Sonoma stores in Connecticut.) Robert's company has registered its trademark 'Sprinkles' with the U.S. Patent and Trademark Office for use with bakery goods and retail store services. Recently Josephine opened a neighborhood cupcake shop in New Haven which she calls 'Sprinkles.' Josephine was not aware of Robert's California company or its registered trademark. Based on these facts, if the California company sues Josephine for trademark infringement, which of the following is true: Answers: If Josephine can prove that she was unaware of the California company's registered trademark for 'Sprinkles', the trademark infringement claim will be lost. The California Company can't win this trademark infringement case against Josephine because the California company does not operate any stores in Connecticut. In order to win its trademark infringement claim, the California company will have to prove that it holds a valid trademark to 'Sprinkles' and that the use of 'Sprinkles' by Josephine's cupcake store creates a likelihood of confusion for customers who could think that the New Haven bakery is affiliated with the California company. In order to win its trademark infringement claim, the California company only needs to prove that it holds a valid trademark for 'Sprinkles'.

In order to win its trademark infringement claim, the California company will have to prove that it holds a valid trademark to 'Sprinkles' and that the use of 'Sprinkles' by Josephine's cupcake store creates a likelihood of confusion for customers who could think that the New Haven bakery is affiliated with the California company.

Buyer and Seller have entered into a signed written contract for the purchase of a 100-acre farm in upstate New York. The purchase price is $925,000 with a closing date of November 1. One week before the closing, Seller informs Buyer that he will not sell the farm for less than $1,000,000. Buyer is not happy with this but agrees to the new price and they both sign a written contract at the new price. Under these circumstances

Since the second contract is in writing and signed and it is the last agreement entered into by the parties, the second contract at the $1,000,000 purchase price will be enforced

Which of the following statements is true regarding the Duke Power Company case decided by the U. S. Supreme Court? Answers: The Court's decision in this case was one of the triggers for the U.S. Civil War. The Court's decision in this case invalidated as unconstitutional the employment discrimination provisions of Title VII of the Civil Rights Act of 1964. The Court's decision expanded the employment discrimination protections of Title VII of the Civil Rights Act of 1964 to prohibit disparate impact discrimination. The Court's decision made discrimination illegal in public accommodations.

The Court's decision expanded the employment discrimination protections of Title VII of the Civil Rights Act of 1964 to prohibit disparate impact discrimination.

ohn was enjoying his annual visit to the State Fair and decided this was the year he would ride the mechanical bull known as "Rolling Thunder." He watched other riders try to master the mechanical bull and saw each one of them thrown to the ground within minutes of starting their ride. John paid his $5 admission charge and got onto the bull. Shortly after the ride started, John was thrown off onto a soft pad underneath the bull, landing on his head and shoulders. John suffered a fractured neck. He sued the State Fair to recover damages on a claim of negligence.Which of the following is true: Answers: The State Fair is not required to exercise reasonable care to keep the fairgrounds and rides safe because it is not a for-profit business. The State Fair can make a strong argument that it is not liable under the doctrine of res ipsa loquitor. The State Fair can make a strong argument that it is not liable because being thrown and injured by a mechanical bull ride is an open and obvious danger and John voluntarily assumed that risk. Riding a mechanical bull is not a 'necessary of life,' so John will need to prove that the State Fair was grossly negligent to win his negligence suit. Because the State Fair donates 10% of its revenues to the local homeless shelter, it can not be held liable for injuries suffered by visitors to the fair.

The State Fair can make a strong argument that it is not liable because being thrown and injured by a mechanical bull ride is an open and obvious danger and John voluntarily assumed that risk.

Mocha Magic, Inc. is a Delaware corporation owned by four brothers. The corporation owns a coffee shop in Cheshire, Connecticut called Mocha Magic. To save money, the brothers handled the incorporation of Mocha Magic, Inc. by themselves without using a lawyer. They filed the Certificate of Incorporation with the Delaware Secretary of State and paid the necessary filing fees to form Mocha Magic, Inc. However, they have not adopted any organizational documents for the corporation, they have not elected directors of the corporation and they haven't appointed any officers. To save money on bank fees, they use one of the brother's personal bank account for the business. The coffee supply company regularly delivers large burlap sacks of coffee beans to the shop. Under their written supply contract, Mocha Magic, Inc. is required to pay for the coffee beans within 30 days after the coffee supply company sends an invoice. Mocha Magic, Inc. has fallen behind in its payments to the supply company. A six-month road construction project closed the street on which Mocha Magic is located and, of course, business is suffering. The coffee supplier is tired of sending invoices to Mocha Magic, Inc. without receiving payment and brought a lawsuit to collect the $5,000 due on unpaid invoices for coffee beans. The supplier sued Mocha Magic, Inc. and each of the four brothers individually. Under these circumstances, which of the following is true? Answers: A corporation cannot be legally established without using an attorney, so all four brothers are personally liable for the $5,000 due the coffee supplier. The coffee supplier has a strong legal claim against all four brothers as shareholders of Mocha Magic, Inc. under the doctrine of 'piercing the veil.' Since the road construction project was an unforeseen circumstance, the obligation to pay the $5,000 is unenforceable. The coffee supplier has a strong legal claim against all four brothers because shareholders of a corporation are always personally liable for the obligations of the corporation.

The coffee supplier has a strong legal claim against all four brothers as shareholders of Mocha Magic, Inc. under the doctrine of 'piercing the veil.'

To win a strict product liability action in a state that has adopted Section 402A of the Restatement (Second) of Torts, a plaintiff must prove each of the following elements EXCEPT: Answers: The defendant manufactured or sold a product in a defective condition unreasonably dangerous to the injured person. The defendant failed to use reasonable care in the manufacture or design of the product. The product reached the consumer without substantial change in the condition in which it was sold. The defendant is engaged in the business of selling the product.

The defendant failed to use reasonable care in the manufacture or design of the product.

Susannah decided to trim some branches from the trees in her backyard. She went to the local ACE hardware store and bought a new Craftsman battery-powered chain saw for $85. When she got home she took it out of the box, quickly looked at the instructions and then climbed a ladder to cut off some of the tree branches. She lost her balance and fell off the ladder. The chain saw came into contact with her left hand and cut off two of her fingers (ouch!!). When she was released from the hospital, she consulted an attorney to find out who she could sue to recover for her injuries. Her lawyer told her that he thinks that the chain saw she bought had a design defect because it didn't have a lock-out or 'dead man' switch which would have stopped the chain saw from running when she was not pressing down on the trigger. Assuming this is considered a design defect making the product unreasonably dangerous, who can be held liable for Susannah's injuries under the doctrine of strict product liability? Answers: Only the local ACE hardware store where she bought the chain saw. Susannah can recover against the local Ace Hardware store and the chain saw manufacturer but the maximum she can recover is ten times the retail cost of the item ($850). Only the manufacturer because it has sole responsibility for the design and manufacture of this product. The local ACE hardware store, the chain saw manufacturer and everyone else in the chain of distribution of this product

The local ACE hardware store, the chain saw manufacturer and everyone else in the chain of distribution of this product

A gorilla escapes from a traveling circus, enters a shopping center, and destroys $300,000 worth of property. The business whose property was destroyed sues the circus to recover damages. The circus can prove that it did not act negligently and that it was not at fault in the escape of the gorilla. Under these circumstances: Answers: The plaintiff (business) has a strong legal claim under the doctrine of strict liability for an ultrahazardous activity. The defendant (traveling circus) will definitely win under the doctrine of res ipsa loquitor. The plaintiff (business) will definitely win under the doctrine of collateral estoppel. The plaintiff (business) will definitely lose because the defendant (traveling circus) was not at fault. All of the above

The plaintiff (business) has a strong legal claim under the doctrine of strict liability for an ultrahazardous activity.

Sam is the CEO of Ace Appliance, Inc., which is based in Waco, Texas and manufactures and distributes commercial appliances. Business is booming and Sam needs to expand production by adding a new manufacturing plant. Sam's team has done extensive research and concluded that manufacturing plants are much more expensive to build and operate in the U.S. than overseas. Sam has asked Tom and Susan to present the pros and cons of building the new plant overseas or building it near corporate headquarters in Texas.Tom says, "If we build overseas, we'll be shipping valuable jobs out of the country. Profits shouldn't be the driving factor in this decision, we need to do what's right for all our constitutents including our employees, our community, our state and our country. Going overseas is not just bad for Waco or Texas, it's bad for the country. We should build that new plant here in Waco.Susan responds, "If we want to keep our shareholders happy, we've got to take steps to reduce our overhead while still maintaining the high quality of our product. It's our obligation to do what's best for our shareholders (while still complying with the law and ethical customs)- which, in this case, happens to be what's right for us, too." We should build that new plant overseas. Which of the following is true regarding this case study? Answers: Tom's arguments are consistent with the stakeholder theory of corporate social responsibility. Susan's arguments are consistent with the stakeholder theory of corporate social responsibility. There is no ethical component to this business decision. The best approach is to 'flip a coin.' All of the above.

Tom's arguments are consistent with the stakeholder theory of corporate social responsibility.

f you buy a DVD from Best Buy, you have the legal right to:

Watch as many times as you want and give it away

The campus club, 'Don't Do Drugs!', is launching a public awareness campaign alerting university students to the danger of using illegal drugs. William, the club chairman, recently bought a book at Borders called Drugs the Destroyer (price paid $25). John McBride is the author of the book. William wants to distribute 1,000 copies of the book as part of the public awareness campaign but the club doesn't have enough money to buy them. Instead, William scans the book onto his computer and burns 1,000 CD's of the book. The club freely distributes all of the CD's to students on campus. John McBride sues William for copyright infringement. Which of the following is true? Answers: William is liable for copyright infringement. Because William distributed the CD's for a good cause, he is not liable for copyright infringement. Because William properly purchased the book before he made the CD's, he is not liable for copyright infringement under the First Sale Doctrine. If William had included a notice on the CD's that the copyright in the material belongs to John McBride, then William is not liable for copyright infringement

William is liable for copyright infringement.

Bruce the builder enters into a legally binding enforceable contract with Harriet the homeowner to build an addition on Harriet's house for a total price of $150,000. Bruce had estimated that his cost to complete this project $120,000. After Bruce has put $20,000 worth of materials into Harriet's addition, Harriet wrongfully refuses to let him finish the project. Based on these facts, if Bruce sues, what amount of damages will he be entitled to recover?

$50,000

Which of the following are claims that can arise under the general heading of 'Consent?' Answers: Fraud Mistake Duress Undue influence All of the above

All of the above

Don was driving his truck when a board fell out of his truck bed and onto the road. Alice, who was driving behind Don's truck tried to avoid the board, swerved and struck a telephone pole, causing her severe injuries. Which of the following is correct?

Don is liable to Alice if it is established that he failed to exercise reasonable care in the way he stores the board in his truck or in the way he was driving his truck.

The doctrine of precedent means: Answers: that the victim must testify. that a writ of certiorari must be granted by the U.S. Supreme Court. that the defendant must testify. that judges must base rulings on similar cases within their jurisdiction. the first case to be filed in a court is heard before subsequent cases.

That judges must base rulings on similar cases within their jurisdiction.

Which of the following relationships generally creates an insurable interest? Answers: Husband and wife Doctor and patient Two neighbors All of the above

Husband and wife

Which of the following is a significant factor in deciding whether an employment non-compete agreement is enforceable? Answers: If the employer is listed on the New York Stock Exchange. If the employer is paying the employee a living wage. If the terms of the agreement are reasonably necessary for the protection of the employer. All of the above

If the terms of the agreement are reasonably necessary for the protection of the employer.

In a civil case, the plaintiff must prove the case

by a preponderance of the evidence

Landlord and Tenant have entered into a signed written agreement for the lease of a retail location for Tenant's Mac & Cheese Restaurant. The lease is for a three-year term and requires monthly payments of $3,000. Tenant operates the restaurant for two years but, due to a general economic decline in the region, he closes the business at the end of year 2 (with one year remaining on the lease term). Landlord made no effort to find a new tenant for the space, choosing instead to sue Tenant for $36,000 (the remaining rent due for the last year of the lease term). Assume that if Landlord had made reasonable efforts to find a new tenant for the space, he could have leased the space for the remaining one-year lease term at a monthly rent of $2,000. (You can also assume that Landlord had no other vacant space available at that time to lease.) What amount of damages is Landlord entitled to collect from Tenant? Answers: $36,000 $5,000 $0 $12,000 $34,000

$12,000

Seller is a home owner who entered into a signed written enforceable contract with Buyer for the sale of his Hamden house for $350,000 with the closing scheduled to take place on January 1. One week before the closing Buyer gave Seller notice that he would not buy the house. (Note - the Buyer had no legal basis to terminate the contract.) Seller was able to find a substitute buyer who bought the house for $347,000 (a reasonable price under all the circumstances). However, the closing with the substitute buyer did not take place until April 1. Seller incurred the following costs on the Hamden house for the three months of January February and March: real estate taxes totaling $2,000 and property insurance totaling $300. What damages is Seller entitled to recover from Buyer under these circumstances? Answers: $350,000 $0 $2,300 $5,300 $3,000

$5,300

in a comparative negligence state, if the plaintiff in a negligence Lawsuit is found to be 30 percent negligent, the plaintiff would recover

70% of the damages.

As it applies to minors, which of the following is LEAST likely to be regarded as a necessary? Answers: A concert ticket Food Rent Medical expenses

A concert ticket

Which of the following contracts is not required to be in writing under the Statute of Frauds? Answers: A three-year employment contract. A contract to buy a car for $6,000. A contract to buy a farmhouse for $350,000. A contract to buy a couch for $450. A contract to guarantee a friend's $400 loan.

A contract to buy a couch for $450.

Which of the following is/are required to establish promissory estoppel? Answers: A promise made by the defendant. Plaintiff's reliance on the defendant's promise. Establishing that the only way to avoid injustice is to enforce the promise. All of the above

All of the above

Which of the following has legal effect of an offer? All of the above "I offer to sell you my pen for $1" "I'll sell you my pen for $1" "I'll sell you my pen for $1" and gives the note to Carmen

All of the above

The 'Infancy Doctrine' refers to: Answers: The application of the doctrine of res ipsa loquitor to contracts entered into by minors. The voidability of a contract entered into by a person suffering from mental incapacity. The right of a minor to disaffirm a contract. A parent's legal obligation to pay for the necessities of his/her infant child.

The right of a minor to disaffirm a contract.

Barbara the Buyer entered into a signed written contract on January 2nd with Stella the Seller for the purchase of Stella's 2010 BMW car for the purchase price of $12,500. The contract provided for the closing to take place on January 15th (in other words, the money would be paid by Barbara and the car delivered by Stella on January 15th) On January 3rd Barbara changed her mind about buying stellas car and she phoned Stella to tell her she wouldn't be going ahead with the purchase. Which of the following is true?

Barbara is not legally entitled to terminate this contract

Mary, age 16, buys a car from Friendly Auto Dealers. She pays $1,500 for the car. It turns out that the car is really worth $2,500. Friendly wants to disaffirm the contract and get the car back. Which of the following best describes this situation? Answers: Because Mary is a minor, this contract is void and Friendly can get back the car by paying Mary $1,500. Because Mary has driven the car, the contract cannot be disaffirmed. Because Mary is a minor, this contract is voidable, but only at the option of Mary, not at the option of Friendly. Because Mary is a minor, this contract is void and Friendly can get the car back by paying Mary $2,500. If Friendly disaffirms the contract within a reasonable period of time then the contract can be rescinded by Friendly.

Because Mary is a minor, this contract is voidable, but only at the option of Mary, not at the option of Friendly.

Suzy was thrilled shed just won $5,000 in the instant lottery. Lisa, Suzy's friend, texted Suzy: "Congratulations on your winnings! Thats Gr8! Do you think you could possible part with $300 to help me out? Fido my Lab, desperately need prescription medicine to keep her thyroid problem under control and I just don't have the money for it this month" Suzy immediately texted Lisa back: "No problem Lisa Fido's one of my favorite dogs, sure i can give you $300. Stop by my apartment in the morning." when lisa turned up the next morning Suzy told her she changed her mind and wouldn't be sharing her lottery winnings with Fido. Which of the following statements best describes the situation?

Because of Suzy did not receive anything of legal value in exchange for her promise to share her Lottery winnings with Lisa, this agreement is not enforceable.

Fanny Farmer held an annual auction at her farm to sell tools, animals, and leftover crops. At the auction there was no indication given whether the auction was with or without reserve. Bob bid $50 for a plow at the auction, and no one bid against him. The auctioneer did not accept Bob's bid and stated that the plow would not be sold for such a low price. What is the legal effect of these actions? Answers: The plow could not be withdrawn from the auction. Fanny made an offer to sell the plow by putting it up for auction. Two bids must be made before an auction is deemed to be without reserve. Bob's bid constituted an acceptance which formed a valid contract. Bob's bid was only an offer.

Bob's bid was only an offer.

Bertha entered into a signed written contract with Verizon Wireless for a free Samsung Galaxy smartphone and a 2-year cell phone plan for the period January 1, 2017 through December 31, 2018 at a monthly cost of $85. The contract contained a termination clause which provided that a termination fee in the amount of $500 would be payable if the customer terminated the contract during the 2-year plan. On December 25, 2017, Bertha's mom gave Bertha her old iPhone X and added her to the family calling plan on AT&T to save Bertha money. Thrilled with her mom's gift, Bertha brought her Verizon phone back to the Verizon store on January 2, 2018 to turn it in and to terminate her Verizon contract. The customer service representative advised Bertha that she would be charged the $500 termination fee in accordance with the contract and he showed her a copy of her contract which she had signed at the bottom and had initialed in the box next to the termination provision. Bertha responded: "Oh, you see, when I read that provision, I thought it said $50 not $500! If I knew there was a $500 termination fee, I never would have signed the contract." If Verizon sues Bertha to recover the $500 termination fee, the likely result is: Answers: Bertha will win on the basis of collateral estoppel. Bertha will be able to rescind the contract under the doctrine of promissory estoppel. Bertha made a unilateral mistake and will not be able to rescind the contract. Because this is a mutual mistake of fact, Bertha can rescind the contract and is not liable for the termination fee.

Bertha made a unilateral mistake and will not be able to rescind the contract.

Billy agreed to build a cedar deck for the Smiths for $7,000 in accordance with a signed written contract. After he began the work, the Smiths asked Billy to add cedar flower boxes around the deck. Billy replied that it would cost an additional $600 to add these flower boxes. The Smiths agreed to the additional cost and they signed a written contract to the effect. Billy finished the deck including the flower boxes and the Smiths agree that he did an excellent job on the project. Which of the following statements best describes the situation?

Bill is entitled to collect $7,600 for the project

Bill was the sole shareholder of Bill's Sports Bar, Inc. This corporation owned and operated a sports bar in New Haven. The business needed a $150,000 working capital loan and New Haven Bank & Trust Company agreed to make this loan but only if Bill agreed to personally guarantee the loan to the corporation. Bill did (orally) agree to guarantee this loan. Bill signed all of the required loan closing documents except the guaranty agreement. With all those loan documents to sign, the guaranty agreement accidentally got overlooked. The bank made the loan to Bill's Sports Bar, Inc. but unfortunately, the business fell on hard times and shut its doors. The corporation owes the Bank $50,000 on the loan. The corporation has no assets. Bill has a personal savings account with $50,000 in it. The Bank sued Bill seeking payment of the $50,000 loan balance claiming that he guaranteed the corporation's loan. Under these circumstances, the likely result is: Answers: Bill would lose under the doctrine of promissory estoppel. Bill would lose under the Leading Object Rule. Bill would lose under the unforeseen circumstances exception to the Statute of Frauds. Bill would win under the Full Performance exception to the Statute of Frauds. Bill would win because an oral guaranty is never enforceable.

Bill would lose under the Leading Object Rule.

Which of the following would be a valid offer? - While Mary was out riding her new $250 bicycle, she saw her friend Betty walking on the sidewalk and shouted out to her: "Betty, you really ought to get a bicycle like mine - they're only $250 you know." - Billy says to Russ, "I've always liked that birdbath in your garden. It would look great in my new flower bed. I'll buy it from you for $100 here and now." - Jane is at Sally's tag sale and says: "This kitchen table isn't worth more than $50." - Jane is a real estate agent, and while looking at a house she says, "I think we can get $500,000 for this place." - Walter says to Sylvia: "What would you take for that necklace you're wearing? It's my wife's birthday tomorrow and I don't have a present yet."

Billy says to Russ, "I've always liked that birdbath in your garden. It would look great in my new flower bed. I'll buy it from you for $100 here and now."

Bob owns an antique shop specializing in antique prints. John, a customer, admires an antique print of the Hartford waterfront that doesn't have a price tag on it. Bob tells John he can have the print for $125. John says, "No thanks, that's way too much! I won't pay that!" The next day, John reconsiders and goes to the shop and tells Bob he'll take it for $125. Which is true at this point? Answers: John's acceptance of the $125 offer is effective only if Bob has not yet revoked his offer. Bob can tell John that the price is now $200. John can force Bob to take the $125 in exchange for the print. Under the mirror image rule, a contract was formed. If $125 is the objectively reasonable value of this print, Bob must accept John's offer to take it for $125.

Bob can tell John that the price is now $200.

Stan is trying to sell his 2005 Nissan car with 100,000 miles on it. He places an ad for it on Craigslist for $3,000. Stan describes the car in the ad as being "well-maintained and in excellent condition." Bob sees Stan's ad and is interested in buying the car. Before buying it, Bob has the car inspected by his mechanic with Stan present when the mechanic says, "Bob, you know, this car has 100,000 miles on it. The timing chain needs to be replaced immediately. Otherwise, it's in excellent condition." Bob buys the car, does not have the timing chain replaced immediately and the following week the timing chain gives out which ruins the engine. The car has to be towed to the mechanic's shop and the repair estimate is $2,000. If Bob sues Stan to recover the $2,000 repair cost, which of the following statements is true? Answers: Bob cannot recover the $2,000 from Stan because even if Stan knew the car needed a new timing chain, Bob didn't rely on Stan's representations with regard to the car's condition. Bob can recover the $2,000 from Stan on the basis of unconscionability. Bob cannot recover the $2,000 from Stan based on the pre-existing duty rule. Either Bob or Stan can rescind the contract based on mutual mistake of fact.

Bob cannot recover the $2,000 from Stan because even if Stan knew the car needed a new timing chain, Bob didn't rely on Stan's representations with regard to the car's condition.

Builder, Inc. agreed to build Susan a storage building for $8,000. After beginning the project, Builder realized that it could not complete the job and make a profit. Builder demanded $9,500 to complete the building. Susan agreed to pay the $9,500. When the project was complete, Susan paid $8,000 to Builder for the job. If Builder sues Susan for the remaining $1,500: Answers: Builder will win because Susan had a pre-existing duty to pay any additional amounts. Builder will lose because there was no legal consideration to support the additional $1,500. Builder will lose unless he can prove that he would have lost money if he only charged $8,000 for the project. Builder will win because there was consideration for the additional $1,500.

Builder will lose because there was no legal consideration to support the additional $1,500.

Seller was trying to sell his house for $340,000. On June 5, 2019, Buyer went to the Post Office and sent Seller a signed written offer to buy Seller's house for $325,000 (which Seller received on June 8, 2019). On June 9, 2019, Seller went to the Post Office and sent a letter to Buyer stating that Seller was willing to sell the house to Buyer for $332,000 (which Buyer received on June 11, 2019). On June 12, 2019, Buyer went to the Post Office and sent Seller a letter stating his agreement to buy the house for $332,000 (which Seller received on June 14, 2019). Under these circumstances, which of the following is true: . Answers: Buyer and Seller entered into a contract on June 14, 2019. Buyer and Seller entered into a contract on June 12, 2019. No contract has been entered into because Seller made a $332,000 counteroffer to Buyer. Buyer and Seller entered into a contract on June 5, 2019. Buyer and Seller entered into a contract on June 9, 2019.

Buyer and Seller entered into a contract on June 12, 2019.

Seller and Buyer were discussing a possible sale of Sellers classic MG car. They orally agreed on a sale of the car at a price if $17,000 with the closing to take place the following day. Seller took out a pad of paper and wrote "This confirms out agreement that Buyer will buy Sellers classic MG car for $17,000 tomorrow". Seller signed this agreement and cashed and handed it to Buyer (who did not sign it). The next day, Buyer came to Seller house with a cashed check for $17,000 and was excited to pick up the car. Seller told Buyer that had changed his mind and wouldn't be selling the car after all. If buyer sues Seller for breach of contract, the most likely result will be:

Buyer will win because this contract is enforceable against the seller under the statute of fraud

Seller and Buyer were discussing a possible sale of Seller's classic MG roadster car. They orally agreed on a sale of the car at a price of $17,000 with the closing to take place the following day. Seller took out a pad of paper and wrote: "This confirms our agreement that Buyer will buy Seller's classic MG roadster car for $17,000 tomorrow." Seller signed this agreement and handed it to Buyer (who did not sign it). The next day Buyer came to Seller's house with a cashier's check for $17,000 and was excited to pick up the car. Seller told Buyer he had changed his mind and wouldn't be selling the car after all. If Buyer sues Seller for breach of contract, the most likely result will be: Answers: This contract is enforceable by Buyer under the leading object rule so Buyer will win. Buyer will win because this contract is enforceable against the seller under the statute of frauds. This contract is unenforceable by either party under the statute of frauds. Buyer will lose. Seller will win because no contract is enforceable unless it is signed by both parties.

Buyer will win because this contract is enforceable against the seller under the statute of frauds.

Buyer agreed to buy Seller's 2-acre parcel of land for $100,000. Buyer sent Seller a standard-form real estate purchase contract containing all the terms and conditions they agreed upon. As they agreed, Buyer sent Seller a $25,000 check as part of the purchase price with the balance to be paid at the closing scheduled to take place on February 1. Seller deposited that check but never got around to signing the contract. Buyer asked Seller for permission to immediately start construction of his dream house on the property and Seller agreed. Buyer had the driveway graded and the concrete foundation poured. A week before the scheduled closing, Seller called Buyer and told him that he had changed his mind and would not sell him the property. Under these circumstances if Buyer sued Seller to enforce this agreement, the likely result is: Answers: Buyer would lose under the doctrine of stare decisis. Buyer would win under the Part Performance by Buyer exception to the Statute of Frauds. Buyer would lose because he has not fully performed under the contract. Buyer would win because the contract price is greater than $5,000. Buyer would win under the leading object rule.

Buyer would win under the Part Performance by Buyer exception to the Statute of Frauds.

Landscaper Larry enters into a legally binding enforceable contract to grade and re-seed Owners back yard for $1,700. Owner pays Landscaper Larry a deposit of $1,000 prior to the start of work. Larry landscaper starts the grading but stops work without completing the job and is in breach of contract. Owner contacts five other landscapers and gets quotes to complete the job. The quotes range from $850 to $975. Owner hires the substitute landscaper with the lowest quote (850) to finish the work. What amount of damages, if any, is Owner entitled to recover. From Landscaper Larry based on these facts?

C

Which of the following terms designates the legal ability of a party to enter into a contract

Capacity

Cynthia was on her way home from classes at 5:00 in the afternoon. She was driving her new 4-wheel drive SUV which was equipped with top-quality snow tires. It had been snowing lightly during the afternoon and the snow had just turned to freezing rain. She was driving very carefully due to the weather and kept her speed down to 10mph. As she came to the traffic light at Mt. Carmel Avenue she gently applied the brakes. Despite her slow speed, the excellent condition of her SUV and her extreme care, Cynthia lost control of her car when it slid on the black ice under the snow. At the same time, Millie was driving carefully in the opposite direction. Having lost control of her car, Cynthia crashed into Millie's car causing significant damage. If Millie sues Cynthia based on claim of negligence to recover for the damage. To her car, which of the following is true?

Cynthia can make a strong argument that under all circumstances she was exercising reasonable care.

Amy, a recent university graduate, needed a car to get to her new job. Dick of Dick's Auto World agreed to make a $5,500 car loan to Amy to finance her purchase of a car if Amy would provide a creditworthy guarantor for the car loan. To help Amy obtain the loan for the car, Ted, a friend, orally told Dick that he would pay the loan if Amy didn't. If Amy stops paying the loan and Dick sues Ted to recover the balance due on the loan, the most likely result would be: Answers: Dick will lose because he was under a pre-existing duty to guaranty the loan Dick will win because Amy's failure to make the loan payments was an unforeseen circumstance. Dick will win because Ted orally agreed to guaranty Amy's loan. Dick will lose because a guaranty agreement is required to be in writing to be enforceable.

Dick will lose because a guaranty agreement is required to be in writing to be enforceable.

Stuart was selling Gary some land he owned in Minnesota. Gary is a keen golfer and stays in shape by swimming at least 3 miles a day. Stuart told Gary that the developer had agreed to build a golf course and swimming pool in the subdivision as part of the development, and that other improvements would be made soon. Stuart knew that none of these improvements existed and none were planned. Gary was thrilled about the planned golf course and pool. Gary believed Stuart and, based on Stuart's statements, Gary bought the land for $100,000 when it was really only worth $20,000 without these improvements. Gary learns of the error and sues Stuart for fraud. Which of the following statements best describes this situation? Answers: Gary cannot recover because the misrepresentation was not of a material fact. Gary can rescind the contract or recover damages because this is clearly fraud. Either party can rescind the contract because this is a case of mutual mistake. Under the doctrine of freedom of contract, Gary cannot recover from Stuart. Gary cannot recover because he suffered no damages as a result of Stuart's misrepresentation.

Gary can rescind the contract or recover damages because this is clearly fraud.

Amy has been trying to purchase Glen's antique desk for some time, but Glen has been reluctant to sell. One evening Glen said to Amy, "Okay, I'll sell the desk for $550." Amy replied, "Thank you, Glen. I accept." The agreement was not reduced to writing, but Glen and Amy did shake hands. Two days later, Amy sent Glen a letter outlining the terms of the agreement, and stating that she would deliver $550 cash according to the agreement within ten days. The letter was signed, "Best regards, Amy." Glen had not signed anything. Later, Amy had second thoughts, and refused to go through with the purchase. Nothing had been exchanged at this point. Under these circumstances: Answers: Glen can enforce the contract against Amy because the statute of frauds is satisfied under this situation. Glen cannot enforce the contract against Amy because he did not sign the letter. Glen can enforce the contract because they shook hands on the agreement. Glen cannot enforce the contract against Amy because of the leading object rule. Glen cannot enforce the contract under the doctrine of promissory estoppel.

Glen can enforce the contract against Amy because the statute of frauds is satisfied under this situation.

Kristin offers to sell land to Ian for $10,000. Ian agrees to buy the land for $10,000 provided this price includes Kristin's 3 tractors in the shed on the land. Kristin tells Ian that this is not acceptable. Ian then says "Okay, I accept your $10,000 offer. It's a deal." Which of the following is true: Answers: Kristin made a counteroffer to Ian. Ian made a counteroffer, that terminated Kristin's offer. Ian made a counteroffer, but Kristin's offer may still be accepted. There is now a valid contract between Kristin and Ian. Under the doctrine of stare decisis, Kristin is obligated to sell the land to Ian for $10,000.

Ian made a counteroffer, that terminated Kristin's offer.

Which of the following is true regarding the infancy doctrine? Answers: It is illegal for a minor to enter into a contract. If a 16-year old is highly educated then she does not have the right to disaffirm a contract. If a minor has entered into a contract, s/he has the right to disaffirm it while a minor and for a reasonable time after turning 18. The right to disaffirm a contract entered into by a minor terminates immediately at the age of 18.

If a minor has entered into a contract, s/he has the right to disaffirm it while a minor and for a reasonable time after turning 18.

Bill Maher stated on The Tonight Show that he would donate $5 million to a charity of Donald Trump's choice if Trump proved that he is not the 'spawn of his mother having sex with an orangutan.' Trump formally accepted Maher's offer by sending a copy of his birth certificate showing that he is indeed "the son of Fred Trump, not an orangutan" and he demanded the $5 million for charity. Maher ignored Trump's demand. Trump then sued Maher for breach of contract. Based on this, which of the following statements is true? Answers: If reasonable people would conclude under these circumstances that Maher was just joking when he made this $5 million offer, then no contract has been formed. Whenever there is an offer and an acceptance, a contract is formed so Maher is contractually obligated to donate $5 million to a charity of Trump's choice. If Maher can prove that he really wasn't intending to make Trump an offer, then Trump will lose. If Trump can prove that he really thought Maher was making him a serious offer, then Trump will be able to enforce this contract. No contract is formed because Trump made Maher a counteroffer by sending him a copy of his birth certificate.

If reasonable people would conclude under these circumstances that Maher was just joking when he made this $5 million offer, then no contract has been formed.

Bill likes Julia and promises to give her $50 on Saturday. Bill actually did give Julia that $50 on Saturday. On Sunday, Bill demands that Julia return the money because it was only an unenforceable gift promise. Is Bill legally entitled to the return of the money? Under these circumstances, the likely result would be: Answers: Julia is not required to return the $50 to Bill. Whether Julia has spent the money or not, she will be required to return the $50 to Bill. If Julia hasn't spent the money yet, she will be required to return the $50 to Bill. Because Bill's promise was not in writing, he cannot recover the $50 from Julia.

Julia is not required to return the $50 to Bill.

Mary says to Jane, "I will sell you my dog Fifi for $100." Jane says to Mary: Great, I love Fifi. It's a deal!" Two days later Mary tells Jane, "I've changed my mind. I'm keeping the dog." Which of the following statements is true: Answers: Jane made a counteroffer to Mary. Mary and Jane entered into a contract which is not affected by Mary's change of mind. Mary can reject Jane's acceptance. Mary made a counteroffer to Jane when she told her that she changed her mind. Mary effectively revoked her offer to sell Fifi to Jane.

Mary and Jane entered into a contract which is not affected by Mary's change of mind.

Mary had been an executive secretary for the CEO of Wentworth, Inc. for more than 20 years. At Mary's retirement party, the CEO expressed his personal appreciation for all Mary's hard work as well as the appreciation of the company. The CEO promised Mary that the company would pay her $1,000 per month for the rest of her life to supplement her social security. Nothing was put in writing about this promise. Despite this generous promise, Mary never received a dime from the Company following her retirement. If Mary sues the company to enforce this promise, which of the following statements is true: Answers: Mary will lose because the CEO's promise is not supported by consideration. If the company had put this promise to Mary in writing, then Mary could have enforced it. Mary will win because the Company is under a pre-existing duty to pay her the $1,000 per month. Since the promise to pay Mary $1,000 per month was an unforeseen circumstance, Mary will be legally permitted to enforce it.

Mary will lose because the CEO's promise is not supported by consideration.

If a buyer is seeking to enforce a real estate contract that does not satisfy the writing requirements of the Statute of Frauds, which of the following legal principles, if successful, would result in the buyer winning this lawsuit and obtaining a remedy? Answers: The part performance exception to the Statute of Frauds. Unforeseen Circumstances. The leading object rule.

The part performance exception to the Statute of Frauds.

Mike, who is 17 years old, bought a motorcycle from Cycle Sellers, Inc. for $3,000. Mike showed the salesman a valid driver's license for identification which accurately reflected his date of birth. One week before his 18th birthday, Mike returned the motorcycle to Cycle Sellers, Inc. and told the salesman he was disaffirming the contract and wanted his $3,000 back. Which of the following is true? Answers: Mike can disaffirm the contract and will receive back his purchase price of $3,000 upon returning the motorcycle to Cycle Sellers. Either party can rescind this contract because Mike was a minor when he entered into the contract. This contract is voidable under the doctrine of unconscionability. Because he is nearly 18, Mike is not entitled to disaffirm the contract. Mike can only disaffirm the contract if it can be objectively shown that he was taken advantage of because of his age.

Mike can disaffirm the contract and will receive back his purchase price of $3,000 upon returning the motorcycle to Cycle Sellers.

Mrs. Jones was pleased to see little 12-year old Johnny Jackson standing outside the local supermarket selling Christmas wreaths for the Boy Scouts. Johnny asked Mrs. Jones if she would like to buy 4 wreaths again this year and Mrs. Jones said yes: "put me down for my usual 4 wreaths, Johnny." Mrs. Jones completed and signed the purchase acknowledgment form. Johnny thanked Mrs. Jones and told her that the money would help the Boy Scout Foundation in its charitable projects. Johnny told Mrs. Jones that he would deliver the wreaths in 3 weeks and would collect the $100 for the wreaths then. When Johnny came to Mrs. Jones' house with the wreaths later that month, Mrs. Jones told him that she had changed her mind and wouldn't buy the wreaths. Which of the following statements is true under these facts: Answers: Mrs. Jones entered into a legally enforceable contract to purchase the wreaths. If Mrs. Jones had notified Johnny that she didn't want to purchase the wreaths within 24 hours after their meeting, this would constitute a valid and timely rescission of the contract. Since Johnny is 12 years old, Mrs. Jones can disaffirm the transaction due to lack of contractual capacity. Mrs. Jones is not legally obligated to purchase the wreaths because the Boy Scout Foundation is a charitable organization. This is an enforceable modification agreement called an accord and satisfaction.

Mrs. Jones entered into a legally enforceable contract to purchase the wreaths.

Bob and Sally are keen supporters of their college varsity sports teams. Bob is convinced that the men's basketball team this year will make it into the Final Four (semi-finalist round of the national tournament). Sally doesn't think the men's basketball team has a strong enough team to do that well this year. Sally is convinced that the women's rugby team will win the national tournament this year. They agree on the following bets: Bet #1: If the men's basketball team this year does make it into the Final Four, Sally will pay Bob $100. If the team does not make it into the Final Four, Bob will pay Sally $100. Bet #2: If the women's rugby team does win the national tournament this year, Bob will pay Sally $250. If the team does not win the national tournament this year, Sally will pay Bob $250.It turns out that the women's rugby team wins the national tournament but the men's basketball team does not make it to the final four this year. Based on this information, which of the following statement(s) is/are true? Answers: Neither of these bets are legally enforceable because gambling contracts are illegal unless specifically authorized by statute. Since the women's rugby team won the national tournament, a Court will enforce the terms of Bet #2 and issue a judgment against Bob in the amount of $250. Because the amounts wagered are less than $500, both these gambling contracts are legally enforceable. If the men's basketball team had made it into the Final Four, a Court would have enforced the terms of Bet #1 issuing a judgment against Sally in the amount of $100.

Neither of these bets are legally enforceable because gambling contracts are illegal unless specifically authorized by statute.

On May 1, 2017, the Dean of the Business School of Boston University sent Professor Smith a letter offering him a job as professor of business law. The letter included all of the details of the appointment including salary, benefits, job responsibility and a start date of August 1, 2017. Professor Smith immediately replied by letter on May 3, 2017: "Great. I accept your offer of employment but the start date needs to be August 8, 2017. My mother is having a serious operation the first week of August and I need to be with her." When Mom's operation was re-scheduled to July, Professor Smith wrote the Dean on June 1, 2017 to say that he would be starting on August 1st. Based on these facts, which of the following statements is true? Answers: A unilateral contract was formed when Mom's operation was re-scheduled. A contract was formed as soon as the Dean received Smith's letter dated May 3, 2017. A contract was formed when Smith mailed his June 1st letter to the Dean. No contract was formed because an advertisement is generally not an offer. No contract was formed because Smith's May 3rd letter was a counteroffer.

No contract was formed because Smith's May 3rd letter was a counteroffer.

Which of the following accurately describes the doctrine of freedom of contract? Answers: People are free to enter into a contract on essentially whatever terms they choose and, in general, courts will enforce a contract in accordance with its terms. A person under the age of 21 is free to walk away from a contract. A contract will only be enforced if a reasonable person would find its terms objectively reasonable in a free market setting. If an item is offered in an advertisement 'for free,' it is considered an offer to enter into a unilateral contract.

People are free to enter into a contract on essentially whatever terms they choose and, in general, courts will enforce a contract in accordance with its terms.

Professor Rogers wants her students to have the benefit of a great new textbook on the law of torts. She is very conscious of the high cost of the textbooks and does not want her student to have to pay an additional $400 for this tort's textbook. Solely for the purpose of helping her students intellectual growth, Professor Rogers makes 40 copies of the first 7 chapters of this textbook, labels it as a Course Pack for "Teaching Purposes Only" and hands the copies out to all her students. (The textbook had a total of 10 chapters). She charges her students $5.00 each to cover the copying costs for this course pack. Under these circumstances, which of the following statements is true?

Professor Rogers has engaged in copyright infringement

my was not paying attention to her driving and crashed into Sally's parked Mercedes doing $4,000 of damage to the car. Amy left her name and contact info on Sally's windshield. When Sally called, they agreed that Amy would pay Sally $4,000 and that Sally would agree not to sue Amy over this incident. They signed a brief contract reflecting this agreement. Under these circumstances: Answers: Sally has provided Legal Value to Amy by agreeing not to sue Amy over this incident. Legal Value isn't required for this type of contract because it is for less than $5,000. Sally has NOT provided Legal Value to Amy because forbearance never constitutes Legal Value. All of the above.

Sally has provided Legal Value to Amy by agreeing not to sue Amy over this incident.

Sally owned a 2,000 acre country estate. She signed a written agreement with Betty, agreeing to sell the house on the property and "a sufficient amount of land surrounding the house to create a park." The closing date called for in the contract was two months after the date the contract was entered into and the price was $400,000. When Sally refused to honor the agreement, Betty sued. Who will prevail and why? Answers: Betty will win based on the mirror image rule. Sally will win because no financing term was included in the agreement. Betty will win because the parties intended to make a contract. Sally will win because the agreement is not reasonably definite.

Sally will win because the agreement is not reasonably definite.

Seller says to Buyer Bob: "I'll sell you my bicycle for $400." Buyer Bob says: "Great, but I can only pay you $200 now. I'll pay you the other $200 next Friday and pick up the bicycle then." Seller responds: "Well, I wouldn't do this for everyone, but since I know you're a man of your word, Bob, I'll take that deal." Buyer Bob and Seller shake hands to show their agreement. Which of the following statements is true: Answers: There is no contract between Buyer Bob and Seller under the mirror image rule. There is no contract between Buyer Bob and Seller because Buyer Bob's initial response was a counter offer. If Buyer Bob changes his mind, he does not have to buy Seller's bike. Seller and Buyer have entered into a contract. If Seller can find a different buyer who will pay him $400 all in cash, he can revoke his offer to Buyer Bob.

Seller and Buyer have entered into a contract.

Seller owned a gentle cow called Rose. Seller had bought Rose as a breeding cow having paid $550 for her. But after a few years with no baby calves, Seller concluded that Rose could not have calved so he decided to sell her to his neighbor for (gasp!) beef. They agreed on a sales price of $80 for rose and signed a contract. The agreed-on price was based on their assumption that Rose was barren and not able to breed. When the buyer came to the farm to pay the sales price and collect Rose, much to their surprise, they discovered that Rose was pregnant. Seller was no longer willing to sell Rose to the buyer for $80 (since a conformed breeder was worth about $1,000). Buyer sues seller to enforce their written sales agreement. Under these circumstances:

Seller will win based on mutual mistake of fact

In order for someone to avoid a contract on the grounds of intoxication, the level of intoxication must have been: Answers: Only high enough that he was able to notice it. At least as high as that of the other party. At or above the legal limit for drunk driving. So great that he didn't comprehend the nature of the agreement he was entering.

So great that he didn't comprehend the nature of the agreement he was entering.

Which of the following statements is/are true regarding non-compete agreements: Answers: All employment non-compete agreements are unenforceable under the doctrine of unconscionability. All non-compete agreements in connection with the sale of a business with a sales price of less than $1,000,000 are unenforceable. Some states have passed statutes that prohibit employment non-compete agreements for lower-paid employees. All of the above.

Some states have passed statutes that prohibit employment non-compete agreements for lower-paid employees.

Steve, who is 17 years old, bought a motorcycle for Cycle Sellers, Inc. for $3,000. Steve showed the salesman a valid driver's license for identification which accurately reflected his date of birth. One week after his 18th birthday, Steve returned the motorcycle to Cycle sellers inc. and told the salesman he was disaffirming the contract and wanted his $3,000 back. Which of the following is true?

Steve can disaffirm the contract because one week is considered reasonable time

Steven says to Bruce: "I'm going out of town this weekend, I'll sell you my ticket to this weekend's football game for $75." Bruce replies: "I'll think it over, Steven, and will let you know later today." One hour later Steven calls Bruce and tells him that his brother's going to be using the football ticket so he's revoking the offer he made to Bruce. Under these circumstances, which of the following statements is correct? Answers: Steven can revoke his offer under these circumstances. Bruce effectively rejected Steven's offer by replying that he would 'think it over.' Bruce can still accept Steven's offer of $75 to buy the football ticket because Steven is obligated to leave his offer open for a reasonable period of time. When Bruce told Steven that he would think it over and let him know later in the day, he was creating a conditional contract - if he didn't notify Steven that he didn't want to buy the ticket by the end of the day, then they had a deal for him to buy that ticket for $75.

Steven can revoke his offer under these circumstances.

34. Which of the following situations would create liability under the traditional common law doctrine of Strict Liability for Ultrahazardous Activities?

Stuart has too much to drink at Leon's sports bar and while driving home crashes into a police station

Gedge has agreed to buy Student's first edition Harry Potter book for $450. They agree to complete the transaction the next day when Gedge will pay Student the $450 and Student will deliver the book to Gedge. There is no signed contract. The next day Student brings the book to Gedge's office as agreed, and Gedge explains that she's changed her mind and will not buy the book. If Student sues Gedge under these circumstances, the likely result would be: Answers: Student would lose because a contract like this must be in writing and signed by both parties. Student would lose under the Part Performance exception to the Statute of Frauds. Student would lose because Gedge (the defendant) didn't sign a contract to buy this book. Student would win under the leading object rule. Student would win because this agreement is not required to be in writing (or signed) to be enforceable.

Student would win because this agreement is not required to be in writing (or signed) to be enforceable.

Susan sends Joan a written offer to purchase Joan's house on Monday. On Wednesday, Susan is hit by a bus and is killed. Joan send Susan a written acceptance of the offer on Friday (without knowing that Susan has been killed). Which of the following statement best describes the situation:

Susan's death automatically terminated the offer so Joan's 'acceptance' is of no legal effect

Michael was a retired insurance executive living in Miami. Suzy was a barista at Starbucks. They met and fell madly in love. Michael had asked Suzy to marry him several times, but she kept saying no because she valued her independence. Michael eventually realized that he had to address Suzy's need for independence. So, over a candlelit dinner he promised Suzy that if she married him, he would take care of her financially for the rest of her life and she'd never have to work again. Well, that made all the difference to Suzy, and they got married on February 14, 2015. Unfortunately, they didn't live happily ever after - Michael filed for divorce in 2018. (Note that neither of them had signed a pre-nuptial agreement.) As part of the divorce proceedings, Suzy is seeking to enforce Michael's promise of financial lifetime support. Under these circumstances, the likely result is: Answers: Suzy would lose on this argument because a promise in consideration of marriage must be in writing to be enforceable. Suzy would win on this argument because the financial value of this promise is greater than $500. Suzy would lose on this argument because of the leading object rule. Suzy would win under the Part Performance exception to the Statute of Frauds. Suzy would lose because divorce cannot be considered an unforeseen circumstance.

Suzy would lose on this argument because a promise in consideration of marriage must be in writing to be enforceable.

Tony raped Jane in the parking lot of Joe's Brew Pub. The DA office prosecuted Tony on rape charges. Subsequently, Jane filed a lawsuit against Tony for money damages. Classify legal action.

The DA's case was a criminal case; Janes lawsuit was a civil case

n the case of The City of Everett v. Estate of Sumstad, the Supreme Court of the State of Washington ruled that: Answers: The Mitchells as high bidder at the auction are entitled to the $32,207 because they genuinely intended to buy the safe as well as its contents. The Mitchells as high bidder at the auction are entitled to the $32,207 because under all the circumstances, a reasonable person would conclude that the auction was for the sale of the safe and its unknown contents. The City of Everett was entitled to the $32,207 under the state law governing ownership of hidden valuables. The Estate of Sumstad and the Mitchells are each entitled to one-half of the $32,207. The Estate of Sumstad was entitled to the $32,207 because the terms of the will govern.

The Mitchells as high bidder at the auction are entitled to the $32,207 because under all the circumstances, a reasonable person would conclude that the auction was for the sale of the safe and its unknown contents.

The Buyer of a car wash business in Wisconsin insisted on putting this clause into the sales contract: "The Seller will not own or be employed by a car wash business for twenty years anywhere in the US, the continent of North America or anywhere else on Earth" Seller was anxious to retire and move down to Florida with the $150,000 sales proceeds so he reluctantly agreed to these terms. But after six months of playing golf, the Seller got bored and opened a small car wash in Boca Raton, Florida. Buyer found out about Seller's Florida car wash business and sued to enforce the non-compete agreement. Under these circumstances, which of the following best describes the situation?

The Non compete agreement is unenforceable because it is not reasonable as to time or geographic area.

n the case of Mesaros v. U.S. Mint, which of the following statements best describes the legal analysis in the Court's decision: Answers: - The Mint was liable to the Mesaros for breach of contract because the Mint didn't satisfy the commemorative coin orders on a 'first come, first served' basis. - The Mesaros made a counteroffer by sending in their order form which the Mint accepted so the Mint is liable to the Mesaros for breach of contract. - The Mint was liable to the Mesaros for breach of contract because a contract was formed when the Mesaros sent in their order form. - The Mint is not liable to the Mesaros because the Mint is an instrumentality of the government so it can't be sued. - The advertisement received by the Mesaros was a solicitation to make an offer to buy. It was not an offer to sell the coins by the Mint.

The advertisement received by the Mesaros was a solicitation to make an offer to buy. It was not an offer to sell the coins by the Mint.

Contractor agreed to build a front porch for the Smiths for $6,000. After he began the work, the Smiths asked him to add window boxes and an old-fashioned swing to the porch. Contractor replied he would, but it would cost an additional $300. The Smiths agreed to the additional cost and asked Contractor to proceed with the additional work. All of these agreements were in writing and signed by both parties. Under these circumstances: Answers: The agreement to pay an additional $300 is enforceable because it was an unforeseen circumstance. The agreement to pay an additional $300 is NOT enforceable because modification agreements are never enforceable. The agreement to pay an additional $300 is enforceable. It is supported by consideration. The agreement to pay an additional $300 is NOT enforceable because it lacks consideration.

The agreement to pay an additional $300 is enforceable. It is supported by consideration.

Bill and Stella have entered into a written signed contract in which Bill has agreed to buy Stella's antique vase for $1,000. The closing is scheduled to take place two weeks after the signing of the contract. Three days before the scheduled closing, Stella dies. Because of Stella's death the following Answers: The contract is legally binding on Stella's estate which is bound to complete the transaction in accordance with the signed contract. The contract would only be terminated if both Bill and Stella were to die. Since Stella had personally signed the contract, the contract is terminated by her death. Death of either party to a contract always terminates the contract.

The contract is legally binding on Stella's estate which is bound to complete the transaction in accordance with the signed contract.

To win a claim of trademark infringement against a defendant, the trademark holder must establish

The defendants use of trademark creates a likelihood of confusion for customers as to which company is responsible for the product or service

In the Alaska Packers Case, the fishermen had entered into a written contract with the fishing company to sail from California to Alaska working as fishermen during the salmon-fishing season. They agreed to perform all standard duties of fishermen during the trip for $50 for the season plus two cents for each salmon caught. Shortly after arriving in Alaska, the fishermen stopped work and demanded an additional $50 to continue performing their duties as fishermen or they would return to California. The fishing company reluctantly agreed to their demands. When the season was completed, the fishing company paid each fisherman $50 (plus two cents per salmon caught). The fishermen sued for the promised $100. In this case, the Ninth Circuit Court of Appeals held that: Answers: The fishermen lost because the second contract was not signed by the fishing company. The fishermen won based on the unforeseen circumstances exception to the pre-existing duty rule. The fishermen lost because the fishing company's promise to pay $100 was not supported by consideration. The fishermen won based on the doctrine of promissory estoppel.

The fishermen lost because the fishing company's promise to pay $100 was not supported by consideration.

John rented an off-campus apartment signing a one-year lease with the landlord. John was 17 years old when he signed the lease. Shortly after signing the lease, John turned 18 and moved into the apartment. He dropped his rent check off with his landlord for the first four months of the lease but then he stopped paying because he had moved in with his girlfriend. If the landlord sues John to enforce the lease, what is the most likely result? Answers: The landlord will win because John ratified the lease by making four lease payments after turning 18. The landlord will lose because John was a minor when he signed the lease. The landlord will win because John signed the lease. The landlord will lose because housing is a necessary of life.

The landlord will win because John ratified the lease by making four lease payments after turning 18.

Contractor and Owner have entered into a signed written contract in which Contractor agrees to excavate a cellar at Owner's property at a price of $2,500. Shortly after Contractor begins the work, he encounters solid rock which makes it a much more expensive job. Neither Contractor nor Owner had any way of knowing of this problem beforehand. Contractor tells Owner about this problem and assures him he can still get the job done but it will cost an extra $500. Owner agrees to this contract modification and they both sign a written modification agreement. Under these circumstances: Answers: The modification agreement would NOT be enforceable under the pre-existing duty rule. The modification agreement would be enforceable under the leading object rule. The modification agreement would be enforceable under the unforeseen circumstances exception to the pre-existing duty rule. Neither contract is enforceable under the doctrine of promissory estoppel.

The modification agreement would be enforceable under the unforeseen circumstances exception to the pre-existing duty rule.

Bob and Rick entered into a signed written contract for Rick to cut down and remove all the trees on the southeast side of Bob's lot at a cost of $750 for each large tree removed and $500 for each small tree removed. Rick completed the job and presented a bill for $3,250 to Bob (for 3 large trees and 2 small trees). Bob tells Rick that all 5 of the trees were small trees so he inly owes him $2,500. Bob and Rick cannot agree on how much is owed. (There was no definition of small or large tree in the contract). They choose to form a new agreement for Bob and Rick to pay an agreed-upon amount for $2,800. Bob pays $2,800 to Rick. Under these circumstances, which of the following statements is true:

The new agreement is enforceable as an accord and satisfaction

Sandy sends her friend Bob an email on Monday offering to sell Bob her 10-speed bike for $150. On Tuesday, Sandy is in a tragic car accident and is instantly killed. Without knowing about the car accident, Bob sends Sandy an email on Wednesday accepting her offer. Which of the following statements best describes the situation: Answers: A contract has been formed under the Subjective Theory of Contracts. Since Bob was unaware of the tragic car accident, his acceptance is effective and a contract has been formed. There is no contract under the mirror image rule. A contract has been formed by offer and acceptance. The offer to sell the bicycle was terminated by operation of law upon Sandy's death.

The offer to sell the bicycle was terminated by operation of law upon Sandy's death.

Amanda, a recent college graduate, needed a car to get to her new job. Tom of Toms auto World agreed to make a $7,500 car loan to Amanda to finance her purchase of a car is Amanda would provide creditworthy guaranty for the car loan. To held Amanda, get the car loan, Ted (Amanda's good friend) orally told Tom that he would pay the loan if Amanda didn't. If Amanda stops paying the loan and Tom sues Ted to recover the balance due on the loan, the most likely result would be:

Tom will lose because a guarantee agreement is required to be in writing to be enforceable

The Honda car dealership places a full page ad in the newspaper with picture and prices for three models of Honda. Due to an error on the part of the newspaper, the price of the 2014 Honda Accord is listed as $200 instead of $20,000. Bill sees the ad while drinking his morning coffee and rushes down to the dealership and says to the first salesman he sees: "i accept your offer to sell a 2014 Honda Accord for the advertised price of $200." Which of the following is true:

Under the common law of contracts the dealership is not legally obligated to sell Bill the car for the advertised price because the advertisement is a solicitation of an offer to buy

Seller and Buyer enter into a signed written contract in which Seller promises to sell his car to Buyer for $8,000 and Buyer promises to buy the car for $8,000 with the transfer to take place on December 1. At the time the contract is signed by both parties, is it supported by consideration? Answers: Consideration is not needed when the contract is signed by both parties. Consideration is not needed when the contract is for a value less than $10,000. No. Buyer has not provided legal value to Seller. Yes. Both Buyer and Seller have provided the other legal value. No. Seller has not provided legal value to Buyer.

Yes. Both Buyer and Seller have provided the other legal value.

The Minnesota legislature passed a law requiring that employers allow each employee adequate time within each four consecutive hours of work to utilize the nearest convenient restroom. This law is a stare decisis. an executive order. an ordinance. a statute.

a statute.

Seller and buyer negotiate for the sale of a house on 10 acres of land. On September 30, they orally agree on a price of $100,000 to be paid as followed: a $10,000 (nonrefundable) payment by October 3 and the $90,000 balance to be paid at the closing to be held on November 30th. Buyer sends seller a letter on October 1 setting forth all of the terms the two had orally agreed on. Buyer enclosed a $10,000 check with the letter. The letter was signed by Buyer only. Seller deposited the check. With Seller permission, Buyer begins construction of a 1,000 square foot addition to the house on October 4th. On November 28th, two days before the scheduled closing, Seller changes his mind and refuses to complete the transaction. At this point, Buyer addition to the house is nearly complete. Buyer sues seller to enforce the contract. Under these circumstances:

all of the above

The government itself prosecutes the wrongdoer in a case involving behavior so threatening that society outlaws it altogether. This kind of case involves civil law. criminal law. procedural law. private law.

criminal law

Professor Smith is feeling generous one Friday afternoon and tells Sarah Student that she will give her $50 on Monday. Sarah would be very pleased to receive this $50 but is not quite sure what to expect. On Monday, Sarah politely reminds Professor Smith of the promised $50. Smith tells Sarah that she doesn't know what got into her on Friday but she has absolutely no intentions of giving Sarah 50 of her hard-earned dollars. If Sarah sues Smith to enforce this promise, the most likely result is that a court would: Answers: require Smith to follow through on the promise if Sarah can present at least three credible witnesses who will confirm Sarah's claim that Smith did, in fact, promise to give Sarah $50. not require Smith to follow through on the promise if Sarah is under the age of 18. not require Smith to follow through on the promise because it was a gift promise. not require Smith to follow through on the promise because it wasn't in writing.

not require Smith to follow through on the promise because it was a gift promise.


Related study sets

Accounting- Introduction to Managerial Accounting Chapter 2

View Set

N4-5 (第19). ~Vると~: Hễ mà ~

View Set

relationship between cost curves

View Set